JEE-Advanced Maths Contest '16

Hello, guys!

This contest is the first JEE contest on Brilliant. The objective of this game is to get well prepared for JEE-Advanced 2016 preparation having fun as well. There is so much to learn from each other, and the contest is an approach to accomplish the same. Each Brilliant user is invited to participate in the contest under the following rules:

  1. I will start by posting the first problem. If there is a user solves it, then (s)he must post a new one.
  2. You may only post a solution of the problem below the thread of problem and post your proposed problem in a new thread. Put them separately.
  3. Please make a substantial comment.
  4. Make sure you know how to solve your problem before posting it. In case there is no one can answer it within 24 hours, then you must post the solution, and you have a right to post another problem.
  5. If the one who solves the last problem does not post his/her problem after solving it within a day, then the one who has a right to post a problem is the last solver before him/her.
  6. The scope of questions is only what is covered under JEE-Advanced syllabus. If you are unaware of the syllabus cover, You can check out the information brochure.
  7. You can use tricks/short methods/specific cases to post the solution of a problem only if they are logically valid.
  8. In the case of any argument or disagreement among the solvers, the decision/judgement is passed to me.
  9. DO NOT ask the answer to the problem. Just post your detailed solution solution along with the answer. It will be highly helpful to gain confidence in your problem solving and rock in JEE.
  10. Proof problems are not allowed.
  11. You can post a problem only from Maths section.

    • Please write the detailed solutions to the problems.

Format your post is as follows:

SOLUTION OF PROBLEM xxx (number of problem) :

[Post your solution here]

PROBLEM xxx (number of problem) :

[Post your problem here]

Please try to post problems from all the spheres of JEE syllabus and share this note so that maximum users come to know about this contest and participate in it. (>‿◠)✌

#Algebra

Note by Sandeep Bhardwaj
5 years, 2 months ago

No vote yet
1 vote

  Easy Math Editor

This discussion board is a place to discuss our Daily Challenges and the math and science related to those challenges. Explanations are more than just a solution — they should explain the steps and thinking strategies that you used to obtain the solution. Comments should further the discussion of math and science.

When posting on Brilliant:

  • Use the emojis to react to an explanation, whether you're congratulating a job well done , or just really confused .
  • Ask specific questions about the challenge or the steps in somebody's explanation. Well-posed questions can add a lot to the discussion, but posting "I don't understand!" doesn't help anyone.
  • Try to contribute something new to the discussion, whether it is an extension, generalization or other idea related to the challenge.
  • Stay on topic — we're all here to learn more about math and science, not to hear about your favorite get-rich-quick scheme or current world events.

MarkdownAppears as
*italics* or _italics_ italics
**bold** or __bold__ bold

- bulleted
- list

  • bulleted
  • list

1. numbered
2. list

  1. numbered
  2. list
Note: you must add a full line of space before and after lists for them to show up correctly
paragraph 1

paragraph 2

paragraph 1

paragraph 2

[example link](https://brilliant.org)example link
> This is a quote
This is a quote
    # I indented these lines
    # 4 spaces, and now they show
    # up as a code block.

    print "hello world"
# I indented these lines
# 4 spaces, and now they show
# up as a code block.

print "hello world"
MathAppears as
Remember to wrap math in \( ... \) or \[ ... \] to ensure proper formatting.
2 \times 3 2×3 2 \times 3
2^{34} 234 2^{34}
a_{i-1} ai1 a_{i-1}
\frac{2}{3} 23 \frac{2}{3}
\sqrt{2} 2 \sqrt{2}
\sum_{i=1}^3 i=13 \sum_{i=1}^3
\sin \theta sinθ \sin \theta
\boxed{123} 123 \boxed{123}

Comments

Here is the first problem for the inauguration of the contest. It's an easy problem. All the best!

Problem 1:

In a sequence of independent trials, the probability of success in one trial is 14\frac 14. Find the probability that the second success takes place on or after the fourth trial.

Ritu Roy has solved this problem at the first place and provided the solution. Thanks!

Sandeep Bhardwaj - 5 years, 2 months ago

Log in to reply

Solution of problem 1

The required probability is

=13C.14.(34)2.14+14C.14.(34)3.14+15C.14.(34)4.14+...=9256.24=2732=_{ 1 }^{ 3 }{ C }.\frac { 1 }{ 4 } { .\left( \frac { 3 }{ 4 } \right) }^{ 2 }.\frac { 1 }{ 4 } \quad +\quad _{ 1 }^{ 4 }{ C }.\frac { 1 }{ 4 } { .\left( \frac { 3 }{ 4 } \right) }^{ 3 }.\frac { 1 }{ 4 } \quad +\quad _{ 1 }^{ 5 }{ C }.\frac { 1 }{ 4 } { .\left( \frac { 3 }{ 4 } \right) }^{ 4 }.\frac { 1 }{ 4 } \quad +\quad ...\infty \\ =\frac { 9 }{ 256 } .24\\ =\boxed { \frac { 27 }{ 32 } }

Ritu Roy - 5 years, 2 months ago

Log in to reply

How did we get that 24? I used the reverse probability method 1 - 3C2.1/4.1/4.3/4 - 3C3.1/4.1/4.1/4 = 27/32

Shivam Mahajan - 5 years, 1 month ago

Problem 10 :

In ΔABC\Delta \text{ABC}, AB=tan1(sin(2))\text{AB} = \tan^{-1}\left(\sin \left(\sqrt{2} \right) \right) and tan(A2)=ln(π)tan(B2)\tan \left(\dfrac{\text{A}}{2} \right) = \ln(\pi) \tan \left(\dfrac{\text{B}}{2} \right)
Then the vertex C\text{C} lies on

a) Ellipse

b) Parabola

c) Hyperbola

d) Straight Line

Ishan Singh - 5 years, 2 months ago

Log in to reply

@Aditya Kumar

I see that you've posted the next problem of contest i.e. Problem 11, but there is no solution of 10th problem by you. Can you clarify on this?

Sandeep Bhardwaj - 5 years, 2 months ago

Log in to reply

I had posted the solution! Some careless mod has deleted it! I can't believe it.

Aditya Kumar - 5 years, 1 month ago

Log in to reply

@Aditya Kumar Ooops, sorry for that! I don't know who did that.

Can you please post that again?

Sandeep Bhardwaj - 5 years, 1 month ago

@Aditya Kumar @Aditya Kumar ,your solution was incorrect.

Saarthak Marathe - 5 years, 1 month ago

Problem 3:

Let ff be a twice differentiable function, such that f(x)=f(x)f''(x)=-f(x) and f(x)=g(x)f'(x)=g(x) , h(x)=[f(x)]2+[g(x)]2h(x)=[f(x)]^2+[g(x)]^2. Given that h(5)=11h(5)=11, evaluate h(10)h(10) .

Vighnesh Shenoy has solved this problem at the first place and provided the solution. Thanks!

Nihar Mahajan - 5 years, 2 months ago

Log in to reply

Problem 51

If (a+ω)1+(b+ω)1+(c+ω)1+(d+ω)1=2ω1(a+\omega)^{-1}+(b+\omega)^{-1}+(c+\omega)^{-1}+(d+\omega)^{-1}=2\omega^{-1} and (a+ω)1+(b+ω)1+(c+ω)1+(d+ω)1=2ω1(a+\omega')^{-1}+(b+\omega')^{-1}+(c+\omega')^{-1}+(d+\omega')^{-1}=2\omega'^{-1} ; where ω\omega and ω\omega' are the complex cube roots of unity, then what is the value of (a+1)1+(b+1)1+(c+1)1+(d+1)1(a+1)^{-1}+(b+1)^{-1}+(c+1)^{-1}+(d+1)^{-1} ?

Anant Badal - 3 years, 6 months ago

Problem 4:

If nN n \in N , evaluate the value of

(n0)+(n4)+(n8)+, \dbinom{n}{0} + \dbinom{n}{4} + \dbinom{n}{8} + \ldots ,

where (nr)=n!r!(nr)! \dbinom{n}{r} =\dfrac{n!}{r!(n-r)!} .

Akul Agrawal is the first person to solve this problem and provide the solution. Thanks!

A Former Brilliant Member - 5 years, 2 months ago

Log in to reply

2n2+1cosnπ4+2n4\frac { { 2 }^{ \frac { n }{ 2 } +1 }\cos { \frac { n\pi }{ 4 } } +{ 2 }^{ n } }{ 4 }

put i, -i,1, -1 in expansion of (1+x)^n and add

Akul Agrawal - 5 years, 2 months ago

(1+x)n=r=0n(nr)xr \displaystyle (1+x)^n = \sum_{r=0}^{n} \dbinom{n}{r} x^r

1m+(1)m+im+(i)m=4,m=4k, 1^{m} + (-1)^{m} + i^{m} + (-i)^{m} = 4 , m = 4k, else it is equal to 0 0

Putting x=1,1,i,i \displaystyle x=1, -1, i, -i and adding, we have,

r=0(n4r)=(1+i)n+(1i)n+2n4\displaystyle \sum_{r=0}^{\infty} \dbinom{n}{4r} = \dfrac{(1+i)^n + (1-i)^n +2^n}{4}

Samuel Jones - 5 years, 2 months ago

Problem 25 :
If p p is an odd prime number, then (2+5)p2p+1 \lfloor ( 2 + \sqrt{5})^{p} \rfloor - 2^{p+1} is always divisible by :

1)2p 1) 2p
2)3p 2) 3p
3)p+1 3) p+1
4)5p 4) 5p

Vighnesh Shenoy has solved this problem at the first place and provided the solution. But feel free to share new approaches. Thanks!

Sarvesh Nalawade - 5 years, 2 months ago

Log in to reply

(2+5)p+(25)p(2+5)p(2+5)p+(25)p+1 (2+\sqrt{5})^{p} + (2-\sqrt{5})^{p} \le (2+\sqrt{5})^{p} \le (2+\sqrt{5})^{p} + (2-\sqrt{5})^{p} + 1
Now,
(2+5)p+(25)pI (2+\sqrt{5})^{p} + (2-\sqrt{5})^{p} \in I
(2+5)p+(25)p+1I (2+\sqrt{5})^{p} + (2-\sqrt{5})^{p} + 1 \in I
(2+5)p=(2+5)p+(25)p \therefore \lfloor (2+\sqrt{5})^{p} \rfloor = (2+\sqrt{5})^{p} + (2-\sqrt{5})^{p}
S(p)=(2+5)p2p+1=(2+5)p+(25)p2p+1 S(p) = \lfloor (2+\sqrt{5})^{p} \rfloor - 2^{p+1} = (2+\sqrt{5})^{p} + (2-\sqrt{5})^{p} - 2^{p+1}
S=2r=0p12(p2r)2p2r5r2p+1 \therefore S =2\displaystyle \sum_{r=0}^{\frac{p-1}{2}}\dbinom{p}{2r}2^{p-2r}5^{r} - 2^{p+1}
S=2p+1+2r=1p12(p2r)2p2r5r2p+1=2r=1p12(p2r)2p2r5r \therefore S = 2^{p+1} + \displaystyle 2\sum_{r=1}^{\frac{p-1}{2}}\dbinom{p}{2r}2^{p-2r}5^{r} - 2^{p+1} = \displaystyle 2\sum_{r=1}^{\frac{p-1}{2}}\dbinom{p}{2r}2^{p-2r}5^{r}
Clearly, S(p) S(p) is divisible by 2 2 and 5 5
S(p)=2r=1p12p2r(p12r1)2p2r5r S(p) = 2\displaystyle \sum_{r=1}^{\frac{p-1}{2}}\frac{p}{2r}\dbinom{p-1}{2r-1}2^{p-2r}5^{r}
Now since p is a prime number, gcd(r,p)=1,r[1,p1] gcd(r,p) = 1 , r \in \left[1,p-1\right]

Therefore, S(p) S(p) is divisible by p.
Since S(p) S(p) is divisible by 2,p,5 2 , p ,5 it is divisible by 2p,5p 2p, 5p

S(5)=1300 S(5) = 1300
S(5) S(5) is not divisible by 15=35 15 = 3\cdot5 and is also not divisible by 6=5+1 6 = 5 + 1

Thus,
S(p) S(p) is not always divisible by p+1 p + 1 and 3p 3p

The correct options are 1 1 and 4 4

A Former Brilliant Member - 5 years, 2 months ago

Problem 36:

Let aa be a positive real number such that a3=6(a+1)a^3 = 6(a + 1) then, find the nature of the roots of x2+ax+a26=0x^2 + ax + a^2 - 6 = 0.

Both Mayank Chaturvedi and Vighnesh Shenoy have provided complete solution this problem. Thanks!

Saarthak Marathe - 5 years, 1 month ago

Log in to reply

A graphical approach.

We have to see the nature of discriminant

a2(4a224)or8a2Now,graphofa36a6=f(a)takestwoturnsata=2and2,whichshowsa(rootoffunction)ispositiveandatleast2rememberf(x)iscontinuous.ata=8thef(a)isnegative.Sorootoff(a)>8.Henceforequationa36a6=0,a>8;a2>8Wehave8a2<0.sotherootsarenonreal{ a }^{ 2 }-(4{ a }^{ 2 }-24)\\ or\quad 8-{ a }^{ 2 }\\ Now,\quad graph\quad of\quad { a }^{ 3 }-6a-6=f(a)\quad \\ takes\quad two\quad turns\quad at\quad a\quad =\sqrt { 2 } \quad and\quad -\sqrt { 2 } ,\quad \\ which\quad shows\quad a(root\quad of\quad function)\quad is\quad positive\quad and\quad at\quad least\quad \sqrt { 2 } \\remember\quad f(x)\quad is\quad continuous.\\ at\quad a=\sqrt { 8 } \quad the\quad f(a)\quad is\quad negative.\\ So\quad root\quad of\quad f(a)>\sqrt { 8 } .\quad \\ Hence\quad for\quad equation\quad { a }^{ 3 }-6a-6=0,\\ a>\sqrt { 8 } ;{ a }^{ 2 }>8\\ We\quad have\quad 8-{ a }^{ 2 }<0.\quad so\\the \quad roots\quad are\quad non\quad real

Mayank Chaturvedi - 5 years, 1 month ago

Δ=(a)24(a26)=243a2=3(8a2) \Delta = (a)^{2} -4(a^{2}-6) = 24-3a^{2} = 3(8-a^{2})
Consider,
f(a)=a36a6 f(a) = a^{3} -6a - 6

f(a)=3a26=3(a22) f'(a) = 3a^{2} - 6 = 3(a^{2}-2)
For 0a2 0 \le a \le \sqrt{2} , f(a) is decreasing, increasing for a2 a \ge \sqrt{2}
f(0)=6<0 f(0) = - 6 < 0
f(2)=22626<0 f(\sqrt{2}) = 2\sqrt{2} -6\sqrt{2} - 6 < 0
f(8)=88686=286<0 f(\sqrt{8}) = 8\sqrt{8}-6\sqrt{8} - 6 = 2\sqrt{8} - 6 < 0

Thus, the positive root of f(a) f(a) is 8 \ge \sqrt{8}
a8 a \ge \sqrt{8}

Δ<0 \Delta < 0
Thus, the roots are not real.

A Former Brilliant Member - 5 years, 1 month ago

Log in to reply

Just completed solving the question and here you post the solution! But I think proof questions are not allowed, isn't it?

Miraj Shah - 5 years, 1 month ago

Log in to reply

@Miraj Shah Oh yes.

Mayank Chaturvedi - 5 years, 1 month ago

@Miraj Shah I am changing the wording of the question ,as suggested by @Vighnesh Shenoy. @Vighnesh Shenoy and @Mayank Chaturvedi,please make changes in your solutions accordingly.

Saarthak Marathe - 5 years, 1 month ago

@Miraj Shah I guess he should have asked if the roots were real or complex.

A Former Brilliant Member - 5 years, 1 month ago

Log in to reply

@A Former Brilliant Member Mayank should post the new question.

A Former Brilliant Member - 5 years, 1 month ago

Log in to reply

@A Former Brilliant Member OK. @Mayank Chaturvedi,please post the next question

Saarthak Marathe - 5 years, 1 month ago

A non-calculus approach.

a36a6=0{a}^{3}-6a-6=0

Let a=b+2/b a=b+2/b

Therefore,(b+2/b)36(b+2/b)6=0 { \left( b+2/b \right) }^{ 3 }-6(b+2/b)-6=0

Simplifying we get that, b66b3+8=0 {b}^{6}-6{b}^{3}+8=0

Therefore, b3=4 {b}^3=4 or 22

Substitute these values to get aa.

That time we see that only one real solution of aa occurs which is, a=21/3+22/3a={2}^{1/3}+{2}^{2/3}

We see that, a26=6/a {a}^{2}-6=6/a

Substituting this value in x2+ax+a26=0{x}^{2}+ax+{a}^{2}-6=0 we get that,

ax2+a2x+6=0a{x}^{2}+{a}^{2}x+6=0

Assume that the roots of these quadratic equation are real, Then using formula for roots for quadratic equations,

x=a±a2242 x=\dfrac { -a\pm \sqrt { { a }^{ 2 }-24 } }{ 2 }

Then substituting the acquired value of aa in this equation we get that xx is a complex number. Hence, our assumption was wrong.

Saarthak Marathe - 5 years, 1 month ago

Log in to reply

Wondering, how did you thought of that a=b+(2/b).

Mayank Chaturvedi - 5 years, 1 month ago

Log in to reply

@Mayank Chaturvedi One way of solving cubic equation of the type ax3+bx+c=0ax^3+bx+c=0 is to take x=d+y/dx=d+y/d and manipulate the value of yy to get a solvable 6th6th degree equation in dd

Saarthak Marathe - 5 years, 1 month ago

Log in to reply

@Saarthak Marathe By solvable, do you mean quadratic type equations with higher degrees?

Mayank Chaturvedi - 5 years, 1 month ago

Log in to reply

@Mayank Chaturvedi yes

Saarthak Marathe - 5 years, 1 month ago

Log in to reply

@Saarthak Marathe Great!!!

Mayank Chaturvedi - 5 years, 1 month ago

Problem 8: Find the locus of centers of the circles which touch the two circles

x2+y2=a2{x}^2+{y}^2={a}^2 and x2+y2=4ax{x}^2+{y}^2=4ax

externally.

Akul Agrawal has solved this problem in the first place and posted the solution. Thanks!

Rohit Ner - 5 years, 2 months ago

Log in to reply

Difference of distance of center of touching circle from the two given circles is constant. Hence equation will be hyperbola with (0,0) and (2a,0) as focii. Vertex is (a,0). difference between distance of center from focii is 2a-a=a.

Hence locus is

(xa)2(a2)2y2(3a2)2=1 \frac { { \left( x-a \right) }^{ 2 } }{ { \left( \frac { a }{ 2 } \right) }^{ 2 } } -\frac { { y }^{ 2 } }{ { \left( \sqrt { 3 } \frac { a }{ 2 } \right) }^{ 2 } } =1

Akul Agrawal - 5 years, 2 months ago

Problem 41

tanx.tan2x.tan3x dx\displaystyle \int tanx. tan2x. tan3x\text{ }\,dx

Saarthak Marathe has posted the solution to the problem.Thanks!

Miraj Shah - 5 years, 1 month ago

Log in to reply

tan3x=tan2x+tanx1tanx.tan2xtanx.tan2x.tan3x=tan3xtanxtan2x\displaystyle tan3x=\frac{tan2x+tanx}{1-tanx.tan2x}\\ tanx.tan2x.tan3x=tan3x-tanx-tan2x

Now integrating we get,

tanx.tan2x.tan3x.dx=lnsec3x3lnsec2x2lnsecx+constant\displaystyle \int tanx.tan2x.tan3x.dx=\frac{ln|sec3x|}{3}-\frac{ln|sec2x|}{2}-ln|secx|+constant

Saarthak Marathe - 5 years, 1 month ago

Log in to reply

Correct!Post the next question.

Miraj Shah - 5 years, 1 month ago

Problem 2:

Which real values of kk exist such that the following system of equations has no solution? {(k+1)x+8y=4kkx+(k+3)y=3k1 \begin{cases} (k + 1)x + 8y = 4k \\ kx + (k + 3)y = 3k - 1 \end{cases}

Nihar Mahajan has solved this problem at the first place and has posted the solution. Thanks!

Ritu Roy - 5 years, 2 months ago

Log in to reply

For the equations to have no solution, they should be parallel lines, hence their x-y coefficients are in proportion but not in proportion when ratio of the constant terms is included.

k+1k=8k+34k3k1\dfrac{k+1}{k} = \dfrac{8}{k+3}\neq \dfrac{4k}{3k-1}

First solving the quadratic from k+1k=8k+3\dfrac{k+1}{k} = \dfrac{8}{k+3} we have k=1,3k=1,3. However when k=3k=3 are substituted in 4k3k1\dfrac{4k}{3k-1} the ratio does not remain consistent. Hence, k=3k=3 is the solution and number of solutions is 11.

(Feel free to correct me)

Nihar Mahajan - 5 years, 2 months ago

Problem 5:

r=17tan2(rπ16)=?\displaystyle \sum _{ r=1 }^{ 7 }{ { \tan }^{ 2 }\left( \frac { r\pi }{ 16 } \right) } =?

Samuel Jones is the first person to solve this problem and provide the solution. Thanks!

Akul Agrawal - 5 years, 2 months ago

Log in to reply

The sum can be written as 8+r=17sec2(rπ16)\displaystyle -8 +\sum_{r=1}^{7} \sec^2 \left(\dfrac{r \pi}{16}\right)

=8+r=171cos2(rπ16)\displaystyle =-8 + \sum_{r=1}^{7} \dfrac{1}{ \cos^2 \left(\dfrac{r \pi}{16}\right)}

Consider the 15th \displaystyle 15^{\text{th}} Chebyshev Polynomials of the Second kind

U15(x)=215k=115(xcos(rπn))\displaystyle U_{15} (x) = 2^{15}\prod_{k=1}^{15} \left(x-\cos \left(\dfrac{r \pi}{n} \right)\right)

Taking logarithm and differentiating two times at x=0x=0, we have

S=8+43=35\displaystyle S = -8 + 43 =\boxed{35}

This way, we can generalize to nn terms also.

Samuel Jones - 5 years, 2 months ago

Problem 6:

Evaluate the sum of the non - real roots of the equation

x4+x35x212x6=0.x^4+x^3-5x^2-12x-6=0.

Aditya Kumar solved the problem at the first place and provided the solution. Thanks!

Samuel Jones - 5 years, 2 months ago

Log in to reply

The equation can be written as (x22x2)(x2+3x+3)=0\left( { x }^{ 2 }-2x-2 \right) \left( { x }^{ 2 }+3x+3 \right) =0

The non- real solutions are: x=12(3i+3)x=\frac { -1 }{ 2 } \left( \sqrt { 3 } i+3 \right) and x=12(3i3)x=\frac { 1 }{ 2 } \left( \sqrt { 3 } i-3 \right) \quad

Hence the sum is: 3-3

Aditya Kumar - 5 years, 2 months ago

Just for enlightening, I'm posting my solution (again).

Let x=t1\displaystyle x = t-1 the equation converts to

t43t32t23t+1=0\displaystyle t^4-3t^3-2t^2-3t+1 = 0

    t23t23t+1t2=0\displaystyle \implies t^2 - 3t -2 -\dfrac{3}{t} + \dfrac{1}{t^2} = 0

Now, let y=t+1t\displaystyle y = t + \dfrac{1}{t}, we have,

y23y4=0\displaystyle y^2 -3y - 4 = 0

    (y4)(y+1)=0\displaystyle \implies (y-4)(y+1) = 0

    (t24t+1)(t2t+1)=0(y=t+1t)\displaystyle \implies (t^2-4t+1)(t^2-t+1) = 0 \quad \left( \because y = t + \dfrac{1}{t} \right)

    (x22x2)(x2+3x+3)=0(t=x+1) \displaystyle \implies (x^2 - 2x -2)( x^2 + 3x + 3) = 0 \quad \left( \because t = x+1 \right)

Only the second factor gives non - real roots since it's discriminant is negative, therefore sum is 3\boxed{-3}

Samuel Jones - 5 years, 2 months ago

The equation factors out as,

(x22x2)(x2+3x+3)=0 ({x}^{2}-2x-2)({x}^{2}+3x+3)=0

Now solve x22x2=0{x}^{2}-2x-2=0 and x2+3x+3=0{x}^{2}+3x+3=0 .

We see that the discriminant of x2+3x+3=0{x}^{2}+3x+3=0 is negative.

Therefore the sum of the non-real roots are -3

Saarthak Marathe - 5 years, 2 months ago

Problem 7:

If (1+x)n=C0+C1x+C2x2+...+Cnxn{ \left( 1+x \right) }^{ n }={ C }_{ 0 }+{ C }_{ 1 }x+{ C }_{ 2 }{ x }^{ 2 }+...+{ C }_{ n }{ x }^{ n }.

Then find a closed of: C02+2C12+3C22+...+(n+1)Cn2{ C }_{ 0 }^{ 2 }+2{ C }_{ 1 }^{ 2 }+3{ C }_{ 2 }^{ 2 }+...+\left( n+1 \right) { C }_{ n }^{ 2 }

Rohit Ner has solved this problem at the first place and posted the solution. Thanks!

Aditya Kumar - 5 years, 2 months ago

Log in to reply

x(1+x)n=C0x+C1x2+C2x3++Cnxn+1x{(1+x)}^{n}=C_0 x+C_1 {x}^2 +C_2 {x}^3+\cdots +C_n {x}^{n+1}

Differentiating both sides, (1+x)n1(nx+1+x)=C0+2C1x+3C2x2++(n+1)Cnxn{(1+x)}^{n-1}(nx+1+x)=C_0 +2C_1 x +3C_2 {x}^2 +\cdots + (n+1)C_n {x}^{n}

Also (1+1x)n=C0+C1x+C2x2++Cnxn{\left(1+\dfrac{1}{x}\right)}^{n}=C_0 + \dfrac{C_1}{x} + \dfrac{C_2}{{x}^2} +\cdots +\dfrac{C_n}{{x}^{n}}

The given series is sum of all constant terms obtained in the expansion of

(1+x)n1((n+1)x+1).(1+1x)n{(1+x)}^{n-1}((n+1)x+1).{\left(1+\dfrac{1}{x}\right)}^{n}

which is same as coefficient of xn{x}^{n} in (1+x)2n1{(1+x)}^{2n-1} i.e. (2n1n)\binom{2n-1}{n}

plus coefficient of xn1{x}^{n-1} in (n+1)(1+x)2n1(n+1){(1+x)}^{2n-1} i.e. (n+1)(2n1n1)(n+1)\binom{2n-1}{n-1}

So the closed form is (2n1n)+(n+1)(2n1n1)\binom{2n-1}{n}+(n+1)\binom{2n-1}{n-1}

Rohit Ner - 5 years, 2 months ago

Problem 9:

(On behalf of Akul Agarwal)

If f(x)=x+01t(x+t)f(t).dt\displaystyle f(x)=x+\int _{ 0 }^{ 1 }{ t(x+t)f(t).dt } , then find the value of the definite integral,

01f(x)dx\int _{ 0 }^{ 1 }{ f(x)dx }

Ishan Singh has provided the complete solution to the problem. Thanks!

Saarthak Marathe - 5 years, 2 months ago

Log in to reply

f(x)=x(1+01tf(t) dt)+01t2f(t) dt \displaystyle f(x) = x \left(1+\int_{0}^{1} t f(t) \ \mathrm{d}t \right) +\int_{0}^{1} t^2 f(t) \ \mathrm{d}t

=Ax+B\displaystyle = Ax +B (say)

    f(x)=x(1+01t(At+B) dt)+01t2(At+B) dt\displaystyle \implies f(x) = x\left(1+\int_{0}^{1} t (At+B) \ \mathrm{d}t \right) + \int_{0}^{1} t^2 (At+B) \ \mathrm{d}t

=x(1+A3+B2)+A4+B3\displaystyle = x \left(1+\frac{A}{3}+\frac{B}{2} \right) + \frac{A}{4} + \frac{B}{3}

    Ax+B=x(1+A3+B2)+A4+B3\displaystyle \implies Ax+B = x \left(1+\frac{A}{3}+\frac{B}{2} \right) + \frac{A}{4} + \frac{B}{3}

Comparing coefficients, we have,

A=6523\displaystyle A = \frac{65}{23}

    01f(t)dt=A1=4223\displaystyle \implies \int_{0}^{1} f(t) dt = A - 1 = \boxed{\dfrac{42}{23}}

Ishan Singh - 5 years, 2 months ago

Problem 11:

If the medians of a ΔABC\Delta ABC make angles α,β,γ\alpha ,\beta ,\gamma with each other, then find the value of:cot(α)+cot(β)+cot(γ)+cot(A)+cot(B)+cot(C)\cot { \left( \alpha \right) } +\cot { \left( \beta \right) } +\cot { \left( \gamma \right) } +\cot { \left( A \right) } +\cot { \left( B \right) } +\cot { \left( C \right) }

Consider α,β,γ\alpha ,\beta ,\gamma to be acute.

Saarthak Marathe is the first person to solve this problem and provide the solution. Thanks!

Aditya Kumar - 5 years, 2 months ago

Log in to reply

The answer is 00.

Saarthak Marathe - 5 years, 2 months ago

Problem 12:

Evaluate:

limx0{sinxx}1{tanxx}\large \displaystyle \lim_{ x\rightarrow 0 }{ \left\{ \frac { sinx }{ x } \right\} }^{ \frac { 1 }{ \left\{ \frac { tanx }{ x } \right\} } }

where,{.} \{ . \} is fractional part function.

Rishabh Cool has provided a complete solution to this problem. Thanks!

Saarthak Marathe - 5 years, 2 months ago

Problem 13:

f2(n+1)f(n)+2f(n+1)=f(n) f^{ 2 }\left( n+1 \right) f\left( n \right) +2f\left( n+1 \right) =f\left( n \right)

Given f(0)=1f\left( 0 \right) =1

Find limn2nf(n)\lim_{n\to\infty} { 2 }^{ n }f\left( n \right)

Ashu Dablo has solved this problem at the first place and has posted the solution. Thanks!

Akul Agrawal - 5 years, 2 months ago

Log in to reply

@Akul Agrawal Simplify the given equation to the form:

f(n) f\left(n \right) = 2(f(n+1))1f2(n+1) \frac {2(f\left (n+1 \right))} { 1 - f^{2}\left (n+1 \right)}

Now we say that for f(n)=tanθf\left(n \right) = \tan \theta , f(n+1)=tanθ2f\left(n+1 \right)=\tan \frac{\theta} {2}

As f(1) =1, from above relation, we can say that f(n)=tanπ2(n+1)f\left(n \right)=\tan \frac{\pi}{2^{(n+1)}} which for n >,>0-> \infty , -> 0

multiply divide by tanπ2(n+1)tan \frac{\pi}{2^{(n+1)}}

now for x>0x->0 tanxx=1\frac {\tan x}{x} =1

So answer is π4 \frac{\pi}{4}

Sorry for my latex!

Ashu Dablo - 5 years, 2 months ago

The equation becomes inconsistent for n=0 . Please check .

Keshav Tiwari - 5 years, 2 months ago

Problem 14 :

If the sum of the first n terms of an AP is cn2 cn^2, then find the sum of the cubes of these n terms.

Saakshi Singh has posted the solution first. Thanks!

Ashu Dablo - 5 years, 2 months ago

Log in to reply

Using sum formula of AP, we get 2cn=2a+(n1)d2cn= 2a + (n-1)d

2cna=a+(n1)d(2cna)3=(a+(n1)d)3. \Rightarrow 2cn - a = a + (n-1)d \\ \Rightarrow (2cn - a)^3 = ( a + (n-1)d)^3 .

We have to find n=1n(a+(n1)d)3=n=1n(2cna)3\displaystyle \sum_{n=1}^{ n} \left( a + (n-1)d \right)^3 =\displaystyle \sum_{n=1}^{ n}(2cn-a)^3 . Nos putting n=1n=1, we get a=ca=c.

So we have sum of the cubes of nn terms

=c3n=1n(2n1)3= c^3 \cdot \displaystyle \sum_{ n=1}^{ n} (2n-1)^3

=c3(n2(2n21)). = c^3 \left( n^2(2n^2 - 1) \right) .

Saakshi Singh - 5 years, 2 months ago

Log in to reply

I've cleaned up your solution for now. But it's very easy to type an equation using latex which you can learn from this note. @Saakshi Singh

Sandeep Bhardwaj - 5 years, 2 months ago

This one is a fairly easy one:

Problem 16:

The maximum value of the function f(x)=2x315x2+36x48f(x) = 2x^{3} - 15 x^{2} + 36x -48

for {xx2+209x x | x^{2} + 20 \leq 9x}

Vignesh Shenoy has posted the correct solution. Thanks!

Ashu Dablo - 5 years, 2 months ago

Log in to reply

x29x+200 x^{2} - 9x + 20 \le 0

(x4)(x5)0x[4,5] (x-4)(x-5) \le 0 \rightarrow x \in \text{[}4,5\text{]}

f(x)=2x315x2+36x48 f(x) = 2x^{3} - 15x^{2} + 36x - 48

Differentiating,

f(x)=6x230x+36 f(x) = 6x^{2} - 30x + 36

f(x)=6(x25x+6) f'(x) = 6(x^{2}-5x+6)

f(x)=6(x2)(x3) f'(x) = 6(x-2)(x-3)

For x[4,5] x \in \text{[}4,5\text{]} , f(x)>0f(x) f'(x) > 0 \rightarrow f(x) is increasing.

f(x)max=f(5)=7 f(x)_{max} = f(5) = 7

A Former Brilliant Member - 5 years, 2 months ago

Problem 18:

Consider the ellipse :
x225+y216=1 \dfrac{x^{2}}{25} + \dfrac{y^{2}}{16} = 1

Let L be the length of perpendicular drawn from the origin to any normal of the ellipse. Find the maximum value of L.

Bonus : Solve it without calculus and generalize for any ellipse

Aditya Kumar has solved this problem at the first place and provided the solution. But feel free to share new approaches. Thanks!

Prakhar Bindal - 5 years, 2 months ago

Log in to reply

Solution to Problem 18:

Consider the point (5cos(θ),4sin(θ))(5\cos(\theta),4\sin(\theta)).

The equation of normal would be: 5xsec(θ)4ycsc(θ)5x\sec(\theta)-4y\csc(\theta)

Hence, distance d=925sec2(θ)+16csc2(θ)d=\left| \frac { 9 }{ \sqrt { 25\sec ^{ 2 }{ \left( \theta \right) } +16\csc ^{ 2 }{ \left( \theta \right) } } } \right|

To maximise dd, we have to minimise the denominator. By minimising the denominator(did that by calculus), the denominator comes out to be 9.

Hence dmax=1\boxed{d_{max}=1}

Aditya Kumar - 5 years, 2 months ago

Log in to reply

Absolutely Correct.

Generalisation for any ellipse x^2 / a^2 + y^2/b^2 = 1

Maximum distance of normal from origin = Modulus (b-a)

Also you could have avoided calculus by using AM Gm in last step by writing sec and cosec in terms of tan and cot

Prakhar Bindal - 5 years, 2 months ago

Problem 26 :
Let na=aaantimes ^{n}a = \underbrace{a^{a^{\cdot^{\cdot^{a}}}}}_{n \text{times}}
f(x)=r=1x(rr) f(x) = \displaystyle \sum_{r=1}^{x} (^{r}r)

Find the last digit of f(15) f(15)

Mayank Chaturvedi has solved this problem at the first place and provided the solution. But feel free to share new approaches. Thanks!

A Former Brilliant Member - 5 years, 2 months ago

Log in to reply

The last digit is 8. Short methods are always welcome. I went to find out last digit of individual terms. Yes!. This looks lengthy but when we note down last terms, last digits of 1,2,4,5,6,10,11,14,15 are easy to find. For rest terms, dividing exponents by 4, using modular arithmetic would do.

Mayank Chaturvedi - 5 years, 2 months ago

Log in to reply

Correct.

A Former Brilliant Member - 5 years, 2 months ago

Problem 39:

Solve

x3dydx=y3+y2y2x2\displaystyle {x}^{3}\frac{dy}{dx}={y}^{3}+{y}^{2}\sqrt{{y}^{2}-{x}^{2}}

Saarthak Marathe - 5 years, 1 month ago

Log in to reply

The answer is xy=c(yy2x2)xy=c(y-\sqrt{y^{2}-x^{2}}) The above equation is a homogeneous equation and can be written of the form dydx=yx3+yx2y2x2\frac{dy}{dx}={\frac{y}{x}}^{3}+{\frac{y}{x}}^{2}*\sqrt{y^{2}-x^{2}} Making the substitution y=vxy=vx and we see that dydx=(v+dvdxx)\dfrac{dy}{dx}=(v+\dfrac{dv}{dx}*x). On simplification we get dvv(v21+vv21)=dxx \frac{dv}{v(v^{2}-1+v\sqrt{v^{2}-1})}=\frac{dx}{x}. Taking v21\sqrt{v^{2}-1} and then Multiplying and dividing by vv21v-\sqrt{v^{2}-1} we have dvv21dvv=dxx\frac{dv}{\sqrt{v^{2}-1}} -\frac{dv}{v}=\frac{dx}{x}. On integrating both sides we have lnvv21v=lnx+c\ln{\frac{v-\sqrt{v^{2}-1}} {v}}=\ln{x}+c. This on substituting back vv gives us the above answer.

Vignesh S - 5 years, 1 month ago

Problem 43 :

Find the remainder when 323232 32^{32^{32}} is divided by 7.
Mayank Chaturvedi has solved this problem with the correct solution.

A Former Brilliant Member - 5 years, 1 month ago

Log in to reply

323(mod7)331(mod7)32310+2(3)310+2(1)99(mod7)323232(9)310+2(1)814(mod7)32\equiv -3 (mod 7)\\ {3}^{3}\equiv -1 (mod 7)\\ {32}^{3*10+2}\equiv {(-3)}^{3*10+2} \equiv (1)*9 \equiv 9 (mod 7)\\ {32}^{32^{32}}\equiv {(9)}^{3*10+2} \equiv (1)*81 \equiv 4 (mod 7)\\

Hence the answer is 4\boxed 4

Saarthak Marathe - 5 years, 1 month ago

Log in to reply

The answer is right, however you used
323232(3232)329324(mod7) 32^{32^{32}} \equiv (32^{32})^{32} \equiv 9^{32} \equiv 4 \pmod{7}

A Former Brilliant Member - 5 years, 1 month ago

Log in to reply

@A Former Brilliant Member Yes i agree with you. Have a check @Saarthak Marathe the answer is a coincidence this time.

Let me try once.

Reference-euler's theorem

φ(7)=6andgcd(32,7)=13261mod(7)............me1Nowwefind3232mod63232232444mod(6)......me2Usingme1andme2323232326x+4324mod(7)32444256=4mod(7)So4istheremainder:)\varphi (7)=6\quad and\quad gcd(32,7)=1\\ { 32 }^{ 6 }\equiv 1mod(7)............me-1\\ Now\quad we\quad find\quad { 32 }^{ 32 }mod\quad 6\\ { 32 }^{ 32 }\equiv { 2 }^{ 32 }\equiv 4^{ 4 }\equiv 4mod(6)......me-2\\ Using\quad me-1\quad and\quad me-2\\ { 32 }^{ { 32 }^{ 32 } }\equiv { 32 }^{ 6x+4 }\equiv { 32 }^{ 4 }mod(7)\\ { 32 }^{ 4 }\equiv 4^{ 4 }\equiv 256=4mod(7)\\ So\quad 4\quad is\quad the\quad remainder\quad :)

Mayank Chaturvedi - 5 years, 1 month ago

Log in to reply

@Mayank Chaturvedi Perfect.

A Former Brilliant Member - 5 years, 1 month ago

Problem 44:

Find the locus of point of intersection of tangents to an ellipse x2a2+y2b2=1 \frac{x^2}{a^2}+\frac{y^2}{b^2}=1 at two points,whose eccentric angles differ by a constant angle α\alpha .

Kunal Verma has provided a complete solution to this problem.Thanks!

Saarthak Marathe - 5 years, 1 month ago

Log in to reply

Point of intersection of tangents at points who's eccentric angles are i i and j j :-

x= a×cosi+j2cosij2 a \times \frac{\cos \frac{i+j}{2}}{\cos \frac{i-j}{2}} and y= b×sini+j2cosij2 b \times \frac{\sin \frac{i+j}{2}}{\cos \frac{i-j}{2}}

Thus x2a2 +y2b2 =sec2ij2\frac{x^2}{a^2} \ + \frac{y^2}{b^2} \ = \sec^2 \frac{i-j}{2}

x2a2 +y2b2 =sec2α2\frac{x^2}{a^2} \ + \frac{y^2}{b^2} \ = \sec^2 \frac{\alpha }{2}

Kunal Verma - 5 years, 1 month ago

Log in to reply

Correct! But prove your 1st statement in your solution and post the next question

Saarthak Marathe - 5 years, 1 month ago

Log in to reply

@Saarthak Marathe The thread should clear out older comments. It took ages to put up that solution. Won't live to see the proof of that uploaded. I'll just mention here that it was obtained by solving the parametric equations of tangents.

Kunal Verma - 5 years, 1 month ago

Problem 45

The coefficient of xn6x^{n-6} in the expansion:

n!× [ x (n0) +(n1)(n0) ] [x2 (n1) +(n2)(n1) ] .... [xn (nn1) +(nn)(nn1) ] n! \times \ [ \ x \ - \frac{\binom{n}{0} \ + \binom{n}{1}}{\binom{n}{0}} \ ] \ [ \frac{x}{2} \ - \frac{\binom{n}{1} \ + \binom{n}{2}}{\binom{n}{1}} \ ] \ .... \ [ \frac{x}{n} \ - \frac{\binom{n}{n-1} \ + \binom{n}{n}}{\binom{n}{n-1}} \ ]

is equal to (xy)× zy \binom{x}{y} \times \ z^y

Find x,y,z if all are integers( x, y and z can be in terms of n n )

Sarvesh Nalawade has provided a complete solution to the problem.

Kunal Verma - 5 years, 1 month ago

Log in to reply

Let S = n!i=1n(xi(ni1)+(ni)(ni1)) n! \prod_{i=1}^{n} ( \frac{x}{i} - \frac{{n \choose i-1} + {n \choose i}}{{n \choose i-1}})

Therefore,S=n!i=1n(xi(n+1i)(ni1)) Therefore, S= n! \prod_{i=1}^{n} ( \frac{x}{i} - \frac{{n+1 \choose i}}{{n \choose i-1}} )

S=n!i=1n(xin+1i) S = n! \prod_{i=1}^{n} ( \frac{x}{i} - \frac{n+1}{i} )

Therefore,S=(xn1)n Therefore, S = (x-n-1)^{n} .

Co-efficient of xn6=(n6)(n+1)6x^{n-6} = {n \choose 6} (n+1)^{6}

Therefore x=n , y=6, z= n+1

Sarvesh Nalawade - 5 years, 1 month ago

Log in to reply

You are required to find the individual values of xx , y y and z z , not the sum ,although it is correct. You may post the next problem after posting the solution.

Kunal Verma - 5 years, 1 month ago

Log in to reply

@Kunal Verma Don't use the variable x for two purposes in the same question.

A Former Brilliant Member - 5 years, 1 month ago

Problem 46

If M=(abcbcacab) M = \begin{pmatrix} a & b & c \\ b & c & a \\ c & a & b \end{pmatrix} is an orthogonal matrix with real entries , what is the minimum value of abc ?

Sarvesh Nalawade - 5 years, 1 month ago

Log in to reply

For an orthogonal matrix M,
MMT=I MM^{T} = I where I is the identity matrix.
Multiplying the matrices on LHS , and comparing with the identity matrix we get,
a2+b2+c2=1 a^{2} + b^{2} + c^{2} = 1
ab+bc+ca=0 ab + bc + ca = 0
From these two equations we get,
a+b+c=±1 a + b + c = \pm 1
f(x)=x3(a+b+c)x2+(ab+bc+ca)xd f(x) = x^{3} - (a+b+c)x^{2} + (ab+bc+ca)x - d where d = abc )

Thus, a,b,c are roots of f(x)=0 f(x) = 0
f(x)=x3(a+b+c)x2d f(x) = x^{3} - (a+b+c)x^{2} - d
Let a+b+c=p a + b + c = p
f(x)=x3px2d f(x) =x^{3} - px^{2} - d
Differentiating,
f(x)=3x22px f'(x) = 3x^{2} - 2px
The roots of f(x)=0 f'(x) = 0 are,
x=0,2p3 x = 0, \dfrac{2p}{3}
For the equation to have 3 real roots, the first turn of the graph should be above x-axis, and second must be below x-axis.

Consider case of p=1 p =1
The roots are x=0,23 x = 0 , \dfrac{2}{3}

f(0)f(23)0 \therefore f(0)f\left(\dfrac{2}{3}\right) \le 0
(d)(427d)0 \left(-d\right)\left(\dfrac{-4}{27}-d\right) \le 0
(d)(d+427)0 \left(d\right)\left(d+\dfrac{4}{27}\right) \le 0
d[427,0] d \in \left[\dfrac{-4}{27},0 \right]
When p=1 p = -1
The roots of f(x)=0 f'(x) = 0 are x=0,23 x = 0 , \dfrac{-2}{3}
f(0)f(23)0 f(0)f\left(\dfrac{-2}{3}\right) \le 0
(d)(427d)0 \left(-d\right)\left(\dfrac{4}{27}-d\right)\le 0
d[0,427] d \in \left[0,\dfrac{4}{27}\right]
Range of d [427,427] \left[\dfrac{-4}{27}, \dfrac{4}{27} \right]
The mininum value occurs when a,b,c a,b,c are a permutation of 23,23,13 \dfrac{2}{3}, \dfrac{2}{3}, \dfrac{-1}{3} .
The maximum value occurs when a,b,c a,b,c are a permutation of 23,23,13 \dfrac{-2}{3} , \dfrac{-2}{3} , \dfrac{1}{3}

A Former Brilliant Member - 5 years, 1 month ago

Log in to reply

Great! Please post the next problem. @Vighnesh Shenoy

Sandeep Bhardwaj - 5 years, 1 month ago

An alternate solution to @Vighnesh Shenoy 's already well written and beautiful solution:

a2+b2+c2=1...(1)ab+bc+ca=0...(2)a^2+b^2+c^2=1...(1) \\ab+bc+ca=0...(2)

Let

a=cosαb=cosβc=cosγa=cos\alpha\\b=cos\beta\\c=cos\gamma

From (1)(1) we can write cos2α+cos2β=sin2γ...(3)cos^2\alpha+cos^2\beta=sin^2\gamma...(3)

Now, using (1)(1) and (2)(2) we can write

cosα+cosβ+cosγ=±1cosα+cosβ=±1cosγcos\alpha+cos\beta+cos\gamma=\pm 1\\ cos\alpha+cos\beta=\pm 1-cos\gamma

Squaring both sides we get

cos2α+cos2β+2cosα.cosβ=1+cos2γ2cosγcos^2\alpha+cos^2\beta+2cos\alpha .cos\beta=1+cos^2\gamma \mp 2cos\gamma

From (3)(3) we can write cosα.cosβ=cos2γcosγcos\alpha .cos\beta=cos^2\gamma \mp cos\gamma

Now multiplying cosγcos\gamma on both sides we get:

cosα.cosβ.cosγ=cos3γcos2γcos\alpha .cos\beta .cos\gamma=cos^3\gamma\mp cos^2\gamma

Critical points of the expression on the right-hand side of the above equation

(1.) {0,23}\{0,\frac{2}{3}\} when cosα.cosβ.cosγ=cos3γcos2γcos\alpha .cos\beta .cos\gamma=cos^3\gamma- cos^2\gamma

(2.) {0,23}\{0,\frac{-2}{3}\} when cosα.cosβ.cosγ=cos3γ+cos2γcos\alpha .cos\beta .cos\gamma=cos^3\gamma+cos^2\gamma

On checking these values, minima occurs at cosγ=23cos\gamma = \frac{2}{3} and the minimum value is 427\frac{-4}{27}, and maximum occur maxima occurs at cosγ=23cos\gamma = \frac{-2}{3} and the maximum value is 427\frac{4}{27}

Miraj Shah - 5 years, 1 month ago

Problem 47 : In a ΔABC \Delta ABC the ratio of side BCBC and AC AC to the circumradius is 2 2 and 32 \dfrac{3}{2} respectively. If the ratio of length of angle bisectors of angle B B to length of angle bisector of angle C C is given by α(α1)βγ \dfrac{\alpha(\sqrt{\alpha}-1)}{\beta\sqrt{\gamma}} find
α+β+γ3 \dfrac{\alpha + \beta + \gamma}{3}

Note :
α,β,γ \alpha, \beta, \gamma are positive integers with gcd(α,β)=gcd(α,γ)=gcd(β,γ=1 gcd(\alpha, \beta ) = gcd(\alpha,\gamma) = gcd( \beta, \gamma = 1

A Former Brilliant Member - 5 years, 1 month ago

Log in to reply

Firstly Using Sine Rule we get,

asinA=bsinB=2R \frac { a }{ \sin { A } } =\frac { b }{ \sin { B } } =2R

Hence, we get A=π2andsinB=34 A = \frac{\pi}{2} and \sin{B} = \frac{3}{4}

Let BC=4k,AC=3k,AB=7k BC = 4k , AC = 3k , AB = \sqrt{7}k

Let BD and CE be the respective angle bisectors.

ADCD=BABCandAEEB=ACBC \frac { AD }{ CD } =\frac { BA }{ BC } \quad and\quad \frac { AE }{ EB } =\frac { AC }{ BC }

AD=374+7kandAE=37k AD=\frac { 3\sqrt { 7 } }{ 4+\sqrt { 7 } }k \quad and\quad AE=\frac { 3 }{ \sqrt { 7 } }k

Then using Pythagoras Theorem , we get values of BD and CE as :

BD=471+7kandCE=627kBD=\frac { 4\sqrt { 7 } }{ 1+\sqrt { 7 } } k\quad and\quad CE=\frac { 6\sqrt { 2 } }{\sqrt{ 7 }} k\quad

Their ratio will be : 7(71)92 \dfrac{7(\sqrt{7} - 1 )}{9\sqrt2}

Hence, α=7,β=9,γ=2 \alpha =7,\quad \beta =9,\quad \gamma =2

Therefore, final answer is 6

Sarvesh Nalawade - 5 years, 1 month ago

Log in to reply

Getting the same thing @Sarvesh Nalawade. I think you should go ahead and post the solution

Miraj Shah - 5 years, 1 month ago

Correct.

A Former Brilliant Member - 5 years, 1 month ago

Last problem for this page:Problem No. '50'

A cevian AQAQ of a equilateral Δ\Delta ABCABC is extended to meet circumcircle at PP. If PB=50PB=50 and PC=45PC=45, find PQPQ upto 3 decimal places.

Deeparaj Bhat has provided the answer

Vignesh S - 5 years, 1 month ago

Log in to reply

Great! This is the last problem of this contest on this note. The solution poster will post the next problem on the continued part of the contest: JEE-Advanced Maths Contest (Continued).

Thanks!

Sandeep Bhardwaj - 5 years, 1 month ago

23.684. I'll post the soln in some time...

A Former Brilliant Member - 5 years, 1 month ago

Log in to reply

Waiting for the solution.

Sandeep Bhardwaj - 5 years, 1 month ago

Log in to reply

@Sandeep Bhardwaj Please provide the complete solution.

Vignesh S - 5 years, 1 month ago

Solution by Deeparaj Bhat:

Extend CPCP to DD such that ΔBDP\Delta BDP is equilateral. Then, BCD=QCPQPC=ABC( angles in the same segment are equal )=60=BDP    ΔDCBΔPCQ    DBPQ=DCPC=1+PBPC(DC=DP+PC=BP+PC)    1PC+1PB=1PQ(BP=DB) \begin{aligned}\angle BCD&=\angle QCP \\\angle QPC&=\angle ABC\quad \left(\because \text{ angles in the same segment are equal }\right)\\&=60^{\circ}\\&=\angle BDP\\\implies \Delta DCB &\sim \Delta PCQ\\\implies \frac{ DB }{PQ}&=\frac{ DC}{PC }\\&=1+\frac{PB}{PC}\quad \left( \because DC=DP+PC=BP+PC \right)\\\implies \frac{1}{PC}+\frac{1}{PB}&=\frac{1}{PQ}\quad \left( \because BP=DB \right)\end{aligned}

Substituting the given values the answer comes out to be 23.684.

Sandeep Bhardwaj - 5 years, 1 month ago

Problem 15:

If N is the number of ways in which 3 distinct numbers can be selected from the set {3, 3^2 ,3^3 ,...... ,3^10 } , so that they form a GP, then find the value of N/5.

Ashu Dablo has solved this problem at the first place and provided the solution. But feel free to share new approaches. Thanks!

Saakshi Singh - 5 years, 2 months ago

Log in to reply

@Saakshi Singh

The problem is similar to having to get number of possible Ap's from the set of {1,2,3..10}

So number of possible Ap's is (8+6+4+2)=20

8 for common dif =1 (1,2,3) , (2,3,4) ........(8,9,10)

6 for common dif =2 (1,3,5) , (2,4,6) ........(6,8,10)

4 for common dif =3 (1,4,7) , (2,5,8) ........(4,7,10)

2 for common dif = 4 (1,5,9) and (2,6,10)

So the answer to your question is 4

Ashu Dablo - 5 years, 2 months ago

Log in to reply

the number of possibilities in that order is 8+6+4+2 but the the set containing these in reverse order can also be in gp ie(4,7,10) &(10,7,4) similar is the case with all other elements so the So number of possible GP's is 2(8+6+4+2)=220=40 so N/5 =8 this was also my solution

Ashwin Kumar - 5 years, 2 months ago

Log in to reply

@Ashwin Kumar We have to SELECT the numbers, not arrange them in order, which is why I said the answer is 4.

Ashu Dablo - 5 years, 2 months ago

My answer is 8 (NoteThe following is a table) Difference in power. No terms. Set(of powers) 1. 8. (123,234,...89 10) 2. 6. (135,246....68 10) 3. 4. (147....47 10) 4. 2. (159,2710) Total = 20 But r can be 3^{1,2,3} or 3^{-1,-2,-3} so N = 2x20 = 40 So N/5 =8

Ashwin Kumar - 5 years, 2 months ago

Problem 17:

z z is a complex number in the complex plane such that (z)0\Im(z) \ne 0 .
If z2+z+1z2z+1 \dfrac{z^{2} + z + 1}{z^{2}-z+1} \in \Re find the value of 10z 10|z|

Details :
(z) \Im(z) denotes imaginary part of z z
\Re denotes real numbers.

Prakhar Bindal has solved this problem at the first place and provided the solution. But feel free to share new approaches. Thanks!

A Former Brilliant Member - 5 years, 2 months ago

Problem 19:

If (1x3)n=r=0narxr(1x)3n2r{ \left( 1-{ x }^{ 3 } \right) }^{ n }=\sum _{ r=0 }^{ n }{ { a }_{ r }{ x }^{ r }{ \left( 1-x \right) }^{ 3n-2r } } then find ara_r.

Saarthak Marathe has solved this problem at the first place and provided the solution. But feel free to share new approaches. Thanks!

Aditya Kumar - 5 years, 2 months ago

Log in to reply

Write (1x3)n=(3x(1x)+(1x)3)n=r=0n3r(nr)(x(1x))r(1x)3n3r {(1-{x}^{3})}^{n} = {(3x(1-x)+{(1-x)}^{3})}^{n}=\sum _{ r=0 }^{ n }{ { 3 }^{ r } } \binom{ n }{ r }{ (x(1-x)) }^{ r }{ (1-x) }^{ 3n-3r }

Saarthak Marathe - 5 years, 2 months ago

Answer is 3r(nr){3}^{r}\cdot \dbinom{n}{r}

Saarthak Marathe - 5 years, 2 months ago

Problem 20:

A series of chords of a parabola y2=4ax{y}^{2}=4ax are drawn so that their projections on the straight line,which is inclined at an angle α\alpha to the axis,are of constant length cc. Find the locus of the midpoints of these chords.

Prakhar Bindal has solved this problem at the first place and provided the solution. But feel free to share new approaches. Thanks!

Saarthak Marathe - 5 years, 2 months ago

Log in to reply

Assume end point of chord in parametric form (at^2 , 2at) and (as^2,2as)

Let The equation of line be y = xtan(alpha)+b

Now find foot of perpendicular from end points onto the line. Using distance formula between them and equate it to c .

Let midpoint be (h,k)

2h = a(t^2 + s^2)

2k = 2a(t+s)

From above two relation find value of t-s and t+s as they will come in expression of distance formula.

Substitute these into that expression and on simplifying further and replacing h and k by x and y.

We obtain locus as

p = alpha (inclination of line)

(y^2 - 4ax) (ycosp+2asinp)^2 + (ac)^2 = 0

Alternate : Find out Length of chord using distance of formula. Then find angle between chord and line by using standard results. And finally take component of length of chord along line and equate to c . That would reduce some of the calculation work .

Credits @Samarth Agarwal for alternate way

Prakhar Bindal - 5 years, 2 months ago

Problem 21

Find the equation of the director circle to the circle circumscribing the quadrilateral formed by the lines in order 2x+3y-2 = 0 , x+y =0 , 2x+5y-1 = 0 , 5x+27y-1 = 0

Samarth Agarwal has solved this problem at the first place and provided the solution. But feel free to share new approaches. Thanks!

Prakhar Bindal - 5 years, 2 months ago

Log in to reply

Circle curcumscribing quad is l1l2+(lambda)l3l4=0 lambda can be found by cofficient of xy=0

Here lambda came out to be - 0.5

So circle is (x-11/6)^2+(y-25/6)^2=698/36 and directer circle is (x-11/6)^2+(y-25/6)^2=698/18

Is this correct.?

Samarth Agarwal - 5 years, 2 months ago

The equation is (x11/6)2+(y25/6)2=349/9 {(x-11/6)}^{2}+{(y-25/6)}^{2}=349/9 . The steps are same as that of Samarth's solution

Saarthak Marathe - 5 years, 2 months ago

PROBLEM 22

Let a(n)=1+12+13++12n1a(n)=1+\dfrac{1}{2}+\dfrac{1}{3}+\cdots+\dfrac{1} {2^n-1} .. Then:

A: a(100)100a(100) \leq 100

B: a(100)>100a(100)>100

C: a(200)100a(200)\leq 100

D: a(200)>100a(200)> 100

For sake of clarification a(n)=r=12n11r\displaystyle a\left( n \right) =\sum _{ r=1 }^{ { 2 }^{ n }-1 }{ \frac { 1 }{ r } }

Aditya Kumar has solved this problem at the first place and provided the solution. But feel free to share new approaches. Thanks!

Samarth Agarwal - 5 years, 2 months ago

Log in to reply

Solution to problem 22:

Clearly a(n)na(n)\leq n. Hence, a(100)100a(100)\leq100

Now, for a(200)a(200), we need to do the following steps: a(n)>1+12+(14+14)+(12n+...+12n)a(n)>1+\frac{1}{2}+\left(\frac{1}{4}+\frac{1}{4}\right)+\left(\frac{1}{2^n}+...+\frac{1}{2^n}\right)

This is an AGP.

Hence, a(200)>(112200)+100>100a(200)>\left(1-\frac{1}{2^200}\right)+100>100.

Therefore options A and D are correct.

Aditya Kumar - 5 years, 2 months ago

Problem 23:

Find the closed form of 0nexdx\int _{ 0 }^{ \infty }{ \left\lfloor \frac { n }{ { e }^{ x } } \right\rfloor dx }

Statutory warning: Do not relate this to Gamma Function.

Vighnesh Shenoy has solved this problem at the first place and provided the solution. But feel free to share new approaches. Thanks!

Aditya Kumar - 5 years, 2 months ago

Log in to reply

I=0nexdx I = \displaystyle \int_{0}^{\infty} \left \lfloor \dfrac{n}{e^{x}} \right \rfloor dx
ex=tdx=dtt e^{-x} = t \rightarrow dx = \dfrac{-dt}{t}
I=01ntdtt I = \displaystyle \int_{0}^{1} \left \lfloor nt \right \rfloor \dfrac{dt}{t}
I=k=1n1knk+1nktdt I = \displaystyle \sum_{k=1}^{n-1} \int_{\frac{k}{n}}^{\frac{k+1}{n}} \dfrac{k}{t} dt
I=k=1n1klog(k+1k) I = \displaystyle \sum_{k=1}^{n-1} k\log\left(\dfrac{k+1}{k}\right)
S=k=1n1[(k+1)log(k+1)klog(k)log(k+1)] S = \displaystyle \sum_{k=1}^{n-1} \left[(k+1)\log(k+1)-k\log(k) - \log(k+1)\right]
S=nlog(n)log(n!) S = n\log(n) - log(n!)

A Former Brilliant Member - 5 years, 2 months ago

Time for an easy question.

Problem. 24 :
Let A1,A2,A3,A4 A_{1}, A_{2}, A_{3}, A_{4} be the areas of four faces of a tetrahedron, and h1,h2,h3,h4 h_{1}, h_{2}, h_{3}, h_{4} be the corresponding altitudes. Give that the volume of tetrahedron is 5 cubic units . Find the minimum value of the expression
(A1+A2+A3+A4)(h1+h2+h3+h4)5! \dfrac{(A_{1} + A_{2} + A_{3} + A_{4})(h_{1}+h_{2}+h_{3}+h_{4})}{5!}

Sarvesh Nalawade has solved this problem at the first place and provided the solution. But feel free to share new approaches. Thanks!

A Former Brilliant Member - 5 years, 2 months ago

Log in to reply

Volume of tetrahedron = Ah3 \dfrac{A \cdot h}{3}
15=A1h1=A2h2=A3h3=A4h4 \therefore 15 = A_{1} \cdot h_{1} = A_{2} \cdot h_{2} = A_{3} \cdot h_{3} = A_{4} \cdot h_{4}

We have to minimize :

S=15(A1+A2+A3+A4)(1A1+1A2+1A3+1A4)5! S = \dfrac{15(A_{1}+A_{2}+A_{3}+A_{4})\left(\dfrac{1}{A_{1}} + \dfrac{1}{A_{2}} + \dfrac{1}{A_{3}} + \dfrac{1}{A_{4}}\right)}{5!}
Using AM GM ineqaulity :
i=14Ai4(A1A2A3A4)14 \displaystyle \sum_{i=1}^{4}A_{i} \geq 4\left({A_{1} \cdot A_{2} \cdot A_{3} \cdot A_{4}}\right)^{\frac{1}{4}}

i=141Ai4(1A1A2A3A4)14 \displaystyle \sum_{i=1}^{4} \dfrac{1}{A_{i}} \geq 4\left(\dfrac{1}{A_{1} \cdot A_{2}\cdot A_{3}\cdot A_{4}}\right)^{\frac{1}{4}}
Multiplying these two we get :
i=14Ai×i=141Ai16 \displaystyle \sum_{i=1}^{4}A_{i} \times \displaystyle \sum_{i=1}^{4} \dfrac{1}{A_{i}} \geq 16

Hence,
Smin=15×165!=2 S_{min} = \dfrac{15 \times 16}{5!} = 2

Sarvesh Nalawade - 5 years, 2 months ago

Log in to reply

Correct!
I used the Cauchy-Schwartz inequality instead of the AM-GM.
(A12+A22+A32+A42)(1A12+1A22+1A32+1A42)(A11A1+A21A2+A31A3+A41A4)2 \left(\sqrt{A_{1}}^{2}+\sqrt{A_{2}}^{2}+\sqrt{A_{3}}^{2}+\sqrt{A_{4}}^{2}\right)\left(\dfrac{1}{\sqrt{A_{1}}^{2}} + \dfrac{1}{\sqrt{A_{2}}^{2}} + \dfrac{1}{\sqrt{A_{3}}^{2}} + \dfrac{1}{\sqrt{A_{4}}^{2}}\right) \geq \left(\sqrt{A_{1}}\dfrac{1}{\sqrt{A_{1}}} + \sqrt{A_{2}}\dfrac{1}{\sqrt{A_{2}}} + \sqrt{A_{3}}\dfrac{1}{\sqrt{A_{3}}} + \sqrt{A_{4}}\dfrac{1}{\sqrt{A_{4}}}\right)^{2}
Equality holds when :
A1=A2=A3=A4 A_{1} = A_{2} =A_{3} = A_{4}

A Former Brilliant Member - 5 years, 2 months ago

PROBLEM 27:

csc15+csc165+csc1325+=nπ32\csc ^{ -1 }{ \sqrt { 5 } } +\csc ^{ -1 }{ \sqrt { 65 } } +\csc ^{ -1 }{ \sqrt { 325 } } +\ldots =\frac { n\pi }{ 32 }

If you were given a chance to visit n (its value to be found from above expression) towns, in any way you want irrespective of order. I tell you that you can do so in AB{ A }^{ B } ways.A & B are naturals.Find minimum value of A+B.

A10B8C6D4 A\quad 10\\ B\quad 8\\ C\quad 6\\ D\quad 4.

Vighnesh Shenoy has solved this problem at the first place and provided the solution. But feel free to share new approaches. Thanks!

Mayank Chaturvedi - 5 years, 2 months ago

Log in to reply

S=csc1(5)+csc1(65)+csc1(325) S = \csc^{-1}(\sqrt{5}) + \csc^{-1}(\sqrt{65}) + \csc^{-1}(\sqrt{325}) \ldots
S=sin1(15)+sin1(165)+sin1(1325)+ S = \sin^{-1}\left(\dfrac{1}{\sqrt{5}}\right) + \sin^{-1}\left(\dfrac{1}{\sqrt{65}}\right) + \sin^{-1}\left(\dfrac{1}{\sqrt{325}}\right) + \ldots

S=tan112+tan118+tan1118+S = \tan^{-1}\dfrac{1}{2} + \tan^{-1}\dfrac{1}{8} + \tan^{-1}\dfrac{1}{18} + \ldots
S=k=1tan112k2=k=1tan1(2k+1)tan1(2k1)=π2π4=π4 S = \displaystyle \sum_{k=1}^{\infty} \tan^{-1}\dfrac{1}{2k^{2}} = \sum_{k=1}^{\infty} \tan^{-1}(2k+1) - \tan^{-1}(2k-1) = \dfrac{\pi}{2} - \dfrac{\pi}{4} = \dfrac{\pi}{4}
n=8 \therefore n = 8

I can visit 0 towns, 1 town, 2 towns, and so forth till 8 towns irrespective of order.
Number of ways I can do this = r=08(8r)=28 \displaystyle \sum_{r=0}^{8}\dbinom{8}{r} = 2^{8}
28=44=162 2^{8} = 4^{4} = 16^{2}
Minimum value of A + B = 8.

Next time you give a summation, please give more terms. I am still not sure that the series I used is correct.

A Former Brilliant Member - 5 years, 2 months ago

Log in to reply

I think you mean tan112\tan ^{ -1 }{ \frac { 1 }{ 2 } } in the third line, and not tan114\tan ^{ -1 }{ \frac { 1 }{ 4 } }

A Former Brilliant Member - 5 years, 2 months ago

Correct . And yes i will take care of more terms in the series next time.

Mayank Chaturvedi - 5 years, 2 months ago

Problem 28: (On behalf of Vighnesh Shenoy)

If a\vec a and b\vec b are two vectors such that a=1,b=4|\vec{a}|=1, |\vec b|=4 and ab=2 \vec a \cdot \vec b =2, then find the angle between b\vec b and c\vec c given that c=(2a×b)3b\vec c=\left( 2 \vec a \times \vec b \right) - 3 \vec b.

Mayank Chaturvedi has solved this problem at the first place and provided the solution. But feel free to share new approaches. Thanks!

Sandeep Bhardwaj - 5 years, 2 months ago

Log in to reply

Sir please extend this page by making a new note as it is becoming slower to load the contents and rendering the latex.

Rohit Ner - 5 years, 1 month ago

Log in to reply

Yes sir @Sandeep Bhardwaj. The note is lagging too much and taking too much time to load.

Saarthak Marathe - 5 years, 1 month ago

PROBLEM29PROBLEM \quad 29:

Time for an easy question.

Find the range of β\beta such that (0,2β1) (0,2\beta-1) lies on or inside the triangles formed by the lines. y+3x+2=0 y +3x +2 = 0
3y2x5=0 3y- 2x- 5=0
4y+x14=0 4y+x-14=0

Mayank Chaturvedi - 5 years, 1 month ago

Log in to reply

Simply by drawing the graph I got 43β94 \frac{4}{3} \leq \beta \leq \frac{9}{4}
. The triangle intersects Y-axis at points (0,53) \left(0,\frac{5}{3}\right) and (0,72) \left(0,\frac{7}{2}\right)
532β172 \therefore \frac{5}{3} \leq 2\beta-1 \leq \frac{7}{2}

43β94 \therefore \frac{4}{3} \leq \beta \leq \frac{9}{4}

Sarvesh Nalawade - 5 years, 1 month ago

Log in to reply

Correct

Mayank Chaturvedi - 5 years, 1 month ago

Great! Can you please add the graph for the sake of solution? Thanks!

Sandeep Bhardwaj - 5 years, 1 month ago

Problem 30 :

Let P(x) be a polynomial of degree 11 such that : P(x)=1x+1,0x11 P(x) = \dfrac{1}{x+1} , 0 \le x \le 11

Find the value of P(12) P(12) .

Sarvesh Nalawade - 5 years, 1 month ago

Log in to reply

P(x)(x+1)=1 P(x)(x+1) = 1
P(x)(x+1)1=g(x) P(x)(x+1) - 1 = g(x)
g(x) g(x) has roots 0,1,2,3,411 0,1,2,3,4\ldots 11
g(x)=ax(x1)(x2)(x11) \therefore g(x) = ax(x-1)(x-2)\ldots(x-11)

P(x)(x+1)1=ax(x1)(x2)(x11) P(x)(x+1)-1 = ax(x-1)(x-2)\ldots(x-11)
Substituting x=1 x = - 1 ,
1=a(1)(2)(12)-1 = a(-1)(-2)\ldots(-12)
1=12!a -1 = 12!a
a=112! a = -\dfrac{1}{12!}
P(12)(13)1=112!×12! P(12)(13) - 1 = -\dfrac{1}{12!}\times 12!
P(12)(13)1=1 P(12)(13) - 1 = -1
P(12)=0 P(12) = 0

A Former Brilliant Member - 5 years, 1 month ago

Log in to reply

That's correct !!

Sarvesh Nalawade - 5 years, 1 month ago

Problem 31 :

Given that,
x2+y2+z2=R2 x^{2} + y^{2} + z^{2} = R^{2}

Let,
P=axy+byz P = axy + byz
If Pmax=R2f(a,b) P_{max} = R^{2}f(a,b)
Find f(3,4) f(3,4)

A Former Brilliant Member - 5 years, 1 month ago

Log in to reply

Let x=Rsinαsinβ,y=Rcosβ,z=Rcosαsinβx=R\sin\alpha\sin\beta,y=R\cos\beta,z=R\cos\alpha\sin\beta

P=y(ax+bz)=R22sin2β(asinα+bcosα)Pmax=R22a2+b2f(a,b)=a2+b22f(3,4)=52\begin{aligned} P&=y\left(ax+bz\right)\\&=\dfrac{{R}^2}{2}\sin 2\beta \left(a\sin\alpha+b\cos\alpha\right)\\{P}_{max}&=\dfrac{{R}^2}{2}\sqrt{{a}^2+{b}^2}\\\Rightarrow f(a,b)&=\dfrac{\sqrt{{a}^2+{b}^2}}{2}\\f(3,4)&=\frac{5}{2}\end{aligned}

Rohit Ner - 5 years, 1 month ago

Log in to reply

Correct.

A Former Brilliant Member - 5 years, 1 month ago

I got the answer 5/2. Applying cauchy-schwarz inequality

(a2+b2)(x2y2+y2z2)(axy+byz)(a2+b2)y2(x2+z2)(axy+byz)(a2+b2)y2(R2y2)(axy+byz)R22(a2+b2)(axy+byz)\sqrt { ({ a }^{ 2 }+b^{ 2 })(x^{ 2 }{ y }^{ 2 }+y^{ 2 }z^{ 2 }) } \ge (axy+byz)\\ \sqrt { ({ a }^{ 2 }+b^{ 2 }){ y }^{ 2 }(x^{ 2 }+z^{ 2 }) } \ge (axy+byz)\\ \sqrt { ({ a }^{ 2 }+b^{ 2 }){ y }^{ 2 }(R^{ 2 }-y^{ 2 }) } \ge (axy+byz)\\ \frac { { R }^{ 2 } }{ 2 } \sqrt { ({ a }^{ 2 }+b^{ 2 }) } \ge (axy+byz)

Mayank Chaturvedi - 5 years, 1 month ago

Log in to reply

You guys decide who is going to post next.

A Former Brilliant Member - 5 years, 1 month ago

Problem 32:

The area bounded by the curve y=xx2y=x-{x}^2 and the line y=mxy=mx equals 92\large\frac{9}{2}.

Find the sum of all possible values of mm.

Rohit Ner - 5 years, 1 month ago

Log in to reply

Let the points of intersection between y=xx2y=x-x^2 and y=mxy'=mx be x1x_1 and x2x_2. Therefore x1,x2x_1, x_2 are the roots of the equation x2+(m1)x=0x^2 +(m-1)x=0...(1)(1)

Given:

x1x2yydx=92\displaystyle \int_{x_1}^{x_2} y-y'\, dx =\frac{9}{2}

x1x2x(1m)x3dx=92\displaystyle \int_{x_1}^{x_2} x(1-m)-x^3\, dx=\frac{9}{2}

(1m)(x2x1)(x2+x1)2(x2x1)(x22+x1x2+x12)3=92\large \frac{(1-m)(x_2-x_1)(x_2+x_1)}{2}-\frac{(x_2-x_1)(x_2^2+x_1x_2+x_1^2)}{3}=\frac{9}{2}

Now using equation (1)(1) we can write x2+x1=1mx_2+x_1=1-m; x1x2=0x_1x_2=0 and x2x1=1mx_2-x_1=|1-m|

Therefore finally,

m=2m=-2 or m=4m=4

Therefore, the required answer is 2\boxed{2}

Is this the answer?

Miraj Shah - 5 years, 1 month ago

Log in to reply

Correct. :). Waiting for the next problem.

Rohit Ner - 5 years, 1 month ago

Problem 33:

Two persons AA and BB agree to meet at a place between 1111 to 1212 noon. The first one to arrive waits for 2020 minutes and then leaves. If the time of their arrival be independent and random, what is the probability that AA and BB shall meet?

Miraj Shah - 5 years, 1 month ago

Log in to reply

Just an alternate solution:

Let the time of arrival of AA be denoted along the xaxisx-axis and that of BB along the yaxisy-axis.

Therefore, the required event is denoted graphically be the area enclosed in xy20|x-y|\le 20. Now the ratio of the area of the event to the area of sample space(=3600=3600) gives the answer 59\frac{5}{9}

Miraj Shah - 5 years, 1 month ago

Probability of meeting If the first person comes between 0-40 minutes=2/31/32/3*1/3=2/92/9

Probability of meeting if first person comes between 40-60 minutes=1/31/3

Therefore,total probability=5/95/9

Is this correct? @Miraj Shah

Saarthak Marathe - 5 years, 1 month ago

Log in to reply

Ya! The answer is correct.

Miraj Shah - 5 years, 1 month ago

Problem 34:

Solve:

(x2+y)dydx=6x\displaystyle ({x}^{2}+y)\frac{dy}{dx}=6x

Saarthak Marathe - 5 years, 1 month ago

Log in to reply

We can write the question as follows:

(x2+y)dy=3×2xdx(x^2+y)dy=3\times 2xdx

Now let x2=λx^2=\lambda

(λ+y)=3dλdy(\lambda+y)=3\large \frac{d\lambda}{dy}

We'll do another substitution here, let λ+y=t\lambda+y=t

Therefore,

t+3=3dtdyt+3=3\large \frac{dt}{dy}

Now after rearranging the terms and integrating we get:

3lnx2+y+3=y+c3ln|x^2+y+3| = y+c

Is this correct? @Saarthak Marathe

Miraj Shah - 5 years, 1 month ago

Log in to reply

It is incorrect. You missed a number in your final answer

Saarthak Marathe - 5 years, 1 month ago

Log in to reply

@Saarthak Marathe Got it! it should be x2+y+3x^2+y+3 right?

Miraj Shah - 5 years, 1 month ago

Log in to reply

@Miraj Shah Yes correct!! Post the next question

Saarthak Marathe - 5 years, 1 month ago

Problem 35:

It can be proved that the areas S0,S1,S2,S3,...S_0,S_1,S_2,S_3,... bounded by the xaxisx-axis and the half waves of the curve y=eαxsinβxy=e^{-\alpha x}sin\beta x, x0x\ge 0 form a geometric progression. Find the common ratio of this geometric progression.

Miraj Shah - 5 years, 1 month ago

Log in to reply

Answer: eαπβ{e}^{-\frac{\alpha\pi}{\beta}}

First I wrote the general integral for Sj{S}_{j} and then divided by Sj1{S}_{j-1}

Is it correct? @Miraj Shah

Saarthak Marathe - 5 years, 1 month ago

Log in to reply

The answer is correct! Can you post a clearer picture of your working if possible?

Thanks!

Miraj Shah - 5 years, 1 month ago

Log in to reply

@Miraj Shah The problem is my mobile phone does not have a good camera. I'll tell u what I did. First I wrote the general integral for Sj{S}_{j} and then divided by Sj1{S}_{j-1}

Saarthak Marathe - 5 years, 1 month ago

Log in to reply

@Saarthak Marathe The method is correct! Just add this statement to your solution so that others can get an idea as to what was your approach! Good solution anyways!

Miraj Shah - 5 years, 1 month ago

PROBLEM 37

In how many ways 12 different books can be distributed among 5 children so that 2 get three books each, and 3 get two books each.

Saarthak Marathe and Miraj Shah had solved this question

Mayank Chaturvedi - 5 years, 1 month ago

Log in to reply

(126)(63)(62)(42)(53)=16632000\dbinom{12}{6}\dbinom{6}{3}\dbinom{6}{2}\dbinom{4}{2}\dbinom{5}{3}=16632000

First choose 2 people of 5 which get 3 books each. Select 6 books(3*2=6) out of 12 books for these two 2 people. Then distribution of these 6 books by letting the first person choose 3 books out of 6 and the rest goes to 2nd person. Now for the remaining 3 people,there are 6 books. 1st person selects 2 books out of 6 books and 2nd person chooses 2 out of 4 books and the remaining goes to the 3rd person. I hope it is clear now. @Mayank Chaturvedi

Saarthak Marathe - 5 years, 1 month ago

Log in to reply

Can you please tell what is the numerical figure you are getting after solving your expression?

Miraj Shah - 5 years, 1 month ago

Log in to reply

@Miraj Shah 16632000

Saarthak Marathe - 5 years, 1 month ago

Log in to reply

@Saarthak Marathe OK! Thanks! Waiting for the next question from you...

Miraj Shah - 5 years, 1 month ago

As you asked, your solution is Crystal clear But it would be interesting if we can find out an explanation for solution by @Miraj Shah.Which I feel is as good approach as yours.

Mayank Chaturvedi - 5 years, 1 month ago

The number of ways distributing nn distinct things into rr groups and arranging them among rr people such that s1,s2,s3,...srs_1, s_2,s_3,...s_r denotes the number of things in the respective groups is :

(n!)(r!)(s1!)(s2!)...(sr!)\large \frac{(n!)( r!)}{(s_1!)( s_2!) ...(s_r!)}

Note in the above formula s1s2s3...srs_1\neq s_2\neq s_3\neq ... s_r

For the given question n=12n=12; r=5r=5; s1=s2=3s_1=s_2=3 and s3=s4=s5=2s_3=s_4=s_5=2.

Now, the above formula can be used but, with a slight modification. Since groups are only identified by the number of objects it has, therefore there is no difference between s1s_1 and s2s_2 and there is no difference between s3,s4s_3, s_4 and s5s_5. Hence the number of cases will reduce by a factor of 2!2! with respect to s1s_1 and s2s_2 and by 3!3! with respect to s3,s4s_3, s_4 and s5s_5. Hence, in totality the required answer is:

12!×5!(3!)2(2!)3(2!)(3!)=16632000\large \frac{12!\times 5!}{(3!)^2(2!)^3(2!)(3!)} = 16632000

Is the above answer correct? @Mayank Chaturvedi

Miraj Shah - 5 years, 1 month ago

Problem 38:

Find the number of ways to go from (0,0) (0,0) to (8,7) (8,7) in a rectangle formed by vertices (0,0),(8,0),(0,7),(8,7) (0,0) , (8,0) , (0,7), (8,7) . The person can only move from (i,j) (i,j) to (i+1,j) (i+1,j) OR (i,j+1)(i,j+1) OR (i+1,j+1)(i+1,j+1) in one step.

Saarthak Marathe - 5 years, 1 month ago

Log in to reply

@Saarthak Marathe

No one has solved the problem in the time limit. Can you please post the solution and the next problem?

Thanks!

Sandeep Bhardwaj - 5 years, 1 month ago

Let the person travel dd diagonals in one trip. So the remaining 152d15-2d sides,he travels up or right, which can be done in (152d8d)\dbinom{15-2d}{8-d} ways. The dd diagonals can be placed in (15dd)\dbinom{15-d}{d} ways.

So the total number of ways=d=07(152d8d)(15dd)\displaystyle \sum_{d=0}^{7} \dbinom{15-2d}{8-d}\dbinom{15-d}{d} .

This summation turns out as 108545\boxed {108545}

Saarthak Marathe - 5 years, 1 month ago

Problem no. 40.

An easy question:Find the following (n0)+3(n1)+5(n2)+........+(2n+1)(nn)\dbinom{n}{0}+3\dbinom{n}{1}+5\dbinom{n}{2}+........+(2n+1)\dbinom{n}{n} Miraj Shah has posted the solution to the problem. Thanks!

Vignesh S - 5 years, 1 month ago

Log in to reply

The above question can be written in the following manner:

r=0n(2r+1)(nr)\displaystyle \sum_{r=0}^{n} (2r+1)\dbinom{n}{r} =2r=0nr(nr)+r=0n(nr)=n2n+2n=(n+1)2n =2\displaystyle\sum_{r=0}^{n}r\dbinom{n}{r} +\displaystyle\sum_{r=0}^{n}\dbinom{n}{r} = n2^n +2^n= \boxed{(n+1)2^n}

Miraj Shah - 5 years, 1 month ago

Log in to reply

Please post the next question

Vignesh S - 5 years, 1 month ago

Problem 42:

Evaluate

limn[n1/2n3/2+n1/2(n+3)3/2+n1/2(n+6)3/2+........+n1/2(n+3(n1))3/2]\displaystyle \lim_{n \rightarrow \infty} \left[\frac{{n}^{1/2}}{{n}^{3/2}}+\frac{{n}^{1/2}}{{(n+3)}^{3/2}}+\frac{{n}^{1/2}}{{(n+6)}^{3/2}}+........+\frac{{n}^{1/2}}{{(n+3(n-1))}^{3/2}}\right]

Vighnesh Shenoy has provided solution to this question. Thanks!

Saarthak Marathe - 5 years, 1 month ago

Log in to reply

This problem can be easily solved by using the concept of definite integral as the sum of limits, which is applied as

limnk=0n1n12(n+3k)32=011(1+3x)32dx=[23(1+3x)]10=2326=26=13 \displaystyle \lim_{n \to \infty} \sum_{k=0}^{n-1} \dfrac{n^{\frac{1}{2}}}{(n+3k)^{\frac{3}{2}}} = \displaystyle \int_{0}^{1} \dfrac{1}{(1+3x)^{\frac{3}{2}}}dx = \left[ \dfrac{2}{3(\sqrt{1+3x})}\right]_{1}^{0} = \dfrac{2}{3} - \dfrac{2}{6} = \dfrac{2}{6} = \dfrac{1}{3}

A Former Brilliant Member - 5 years, 1 month ago

Log in to reply

Great! It would be awesome if you can make your solution more detailed. You can post the next problem. Thanks!

Sandeep Bhardwaj - 5 years, 1 month ago

Problem 48 : If 0π2dx(acos2x+bsin2x)2=π(a+b)p(ab)q \displaystyle \int _{ 0 }^{ \frac { \pi }{ 2 } }{ \frac { dx }{ \left( a\cos ^{ 2 }{ x } +b\sin ^{ 2 }{ x } \right) ^{ 2 } } } =\quad \frac { \pi \left( a+b \right) }{ p{ \left( ab \right) }^{ q } } ;

where p and q are positive rational numbers and ab>0 , then find the value of pq

Vighnesh Shenoy has provided a complete solution to this problem.

Sarvesh Nalawade - 5 years, 1 month ago

Log in to reply

I=0π2dx(acos2(x)+bsin2(x)) I = \displaystyle \int_{0}^{\dfrac{\pi}{2}}\dfrac{dx}{(a\cos^{2}(x) + b\sin^{2}(x))}
Put, tan(x)=t \tan(x) = t

I=0dta+bt2=π2abI = \displaystyle \int_{0}^{\infty} \dfrac{dt}{a+bt^{2}} = \dfrac{\pi}{2\sqrt{ab}}
0π2dx(acos2(x)+bsin2(x))=π2(ab)12 \therefore \displaystyle \int_{0}^{\dfrac{\pi}{2}}\dfrac{dx}{(a\cos^{2}(x) + b\sin^{2}(x))} =\dfrac{\pi}{2}(ab)^{\frac{-1}{2}}

For a continuous and differentiable function,
ddyabf(x,y)dx=abyf(x,y)dx\displaystyle \dfrac{d}{dy} \int_{a}^{b} f(x,y)dx = \int_{a}^{b} \dfrac{\partial}{\partial y} f(x,y) dx

Differentiating with respect to a,

0π2adx(acos2(x)+bsin2(x))=a(π2(ab)12) \displaystyle \int_{0}^{\dfrac{\pi}{2}}\dfrac{\partial}{\partial a} \dfrac{dx}{(a\cos^{2}(x) + b\sin^{2}(x))} = \dfrac{\partial}{\partial a} \left( \dfrac{\pi}{2}(ab)^{\frac{-1}{2}}\right)

0π2cos2dx(acos2(x)+bsin2(x))2=π4b12a32 \therefore \displaystyle \int_{0}^{\dfrac{\pi}{2}}\dfrac{-\cos^{2}dx}{(a\cos^{2}(x) + b\sin^{2}(x))^{2}} = \dfrac{-\pi}{4}b^{\frac{-1}{2}}a^{\frac{-3}{2}}

Differentiate with respect to b,

0π2bdx(acos2(x)+bsin2(x))=b(π2(ab)12) \displaystyle \int_{0}^{\dfrac{\pi}{2}}\dfrac{\partial}{\partial b} \dfrac{dx}{(a\cos^{2}(x) + b\sin^{2}(x))} = \dfrac{\partial}{\partial b} \left( \dfrac{\pi}{2}(ab)^{\frac{-1}{2}}\right)

0π2sin2dx(acos2(x)+bsin2(x))2=π4b32a12 \therefore \displaystyle \int_{0}^{\dfrac{\pi}{2}}\dfrac{-\sin^{2}dx}{(a\cos^{2}(x) + b\sin^{2}(x))^{2}} = \dfrac{-\pi}{4}b^{\frac{-3}{2}}a^{\frac{-1}{2}}

Adding both the integrals,

0π2dx(acos2(x)+bsin2(x))2=π4ab(1a+1b) \displaystyle \int_{0}^{\dfrac{\pi}{2}}\dfrac{dx}{(a\cos^{2}(x) + b\sin^{2}(x))^{2}} = \dfrac{\pi}{4\sqrt{ab}}\left(\dfrac{1}{a} + \dfrac{1}{b} \right)

0π2dx(acos2(x)+bsin2(x))2=π(a+b)4(ab)32 \therefore \displaystyle \int_{0}^{\dfrac{\pi}{2}}\dfrac{dx}{(a\cos^{2}(x) + b\sin^{2}(x))^{2}} = \dfrac{\pi(a+b)}{4(ab)^{\frac{3}{2}}}

p=4,q=32 p = 4 , q = \dfrac{3}{2}
pq=6 pq = 6

A Former Brilliant Member - 5 years, 1 month ago

If the equation x^4-Px^2+9=0 has 4 real roots then P lies in the interval:- A} (0,infinity) B} (6,infinity) C} (-infinty,-6)

amrit dubey - 2 years, 11 months ago

If a^b^b^a=a√2, find a^2/b

Kalyanasundaram Perum - 2 years, 10 months ago

is this contest still on ???

Aryan Bansal - 1 year, 7 months ago

Problem 49 :
a,b,c,d a,b,c,d are real numbers such that,
a+2b+3c+4d=15 a + 2b + 3c + 4d = 15
Find the minimum value of
9a+29b+39c+49d 9^{a} + 2\cdot9^{b} + 3\cdot9^{c} + 4\cdot 9^{d}

Vignesh S has posted the solution to this problem. Feel free to add new approaches. Thanks!

A Former Brilliant Member - 5 years, 1 month ago

Log in to reply

The answer is 270 Let f(x)=9a+29b+39c+49df(x)=9^{a}+2\cdot 9^{b}+3\cdot 9^{c}+4\cdot 9^{d} and g(x)=a+2b+3c+4dg(x)=a+2b+3c+4d. Using Lagrange Multipliers we have f(a,b,c,d)x=k×g(a,b,c,d)x\dfrac{\partial{f(a,b,c,d)}}{\partial x}=k \times \dfrac{\partial{g(a,b,c,d)}}{\partial x} where x=a,b,c,dx=a,b,c,d (9a+29b+39c+49d)x=k(a+2b+3c+4d)x\dfrac{\partial{(9^{a}+2\cdot 9^{b}+3 \cdot 9^{c}+4 \cdot 9^{d})}}{\partial x}=k \dfrac{\partial (a+2b+3c+4d)}{\partial x} ln(9)9a=k×1\ln(9) \cdot 9^{a}=k \times1 Similarly doing for b,c,d we have 2ln(9)9b=k×22\ln(9)\cdot 9^{b}=k \times 2 3ln(9)9c=k×33\ln(9)\cdot9^{c}=k \times 3 4ln(9)9d=k×44\ln(9) \cdot 9^{d}=k \times4     a=b=c=d\implies a=b=c=d. So substituting in gg the above condition we get 10a=15    a=1.510a=15\implies a=1.5 Therefore inff(x)=27×(1+2+3+4)=270\inf{f(x)}=27×(1+2+3+4)=270

Vignesh S - 5 years, 1 month ago

Log in to reply

the given expression (S) whose minimum value is to be calculated can be written as

S = 9^a + 9^b + 9^b + 9^c + 9^c + 9^c + 9^d + 9^d + 9^d + 9^d

Now applying AM - GM inequality

S/10 >= ( 9 ^(a+2b+3c+4d) )^1/10

S/10 >= 9^(15/10)

S/10 >= 9^3/2 = 27

Therefore S >= 270

and hence minimum value of S is 270

Aayush Patni - 5 years, 1 month ago

Log in to reply

@Aayush Patni Look who's back from the dead.

Kunal Verma - 5 years, 1 month ago

The answer is correct.

A Former Brilliant Member - 5 years, 1 month ago

@Vignesh S

Please post the next problem(Problem 50) of the contest. And after this problem on this note, the contest will be continued on JEE-Advanced Maths Contest(Continued) note.

Thanks!

Sandeep Bhardwaj - 5 years, 1 month ago
×

Problem Loading...

Note Loading...

Set Loading...